How to evaluate $\int_{5 n \pi}^{\frac{5\pi (n^2 + 1)}{n}}\frac{\frac{8 n^2 x}{25}dx}{((a^2 + b^2) + (a^2 – b^2) cos\frac{2 n x}{5})^2}$

calculusdefinite integralsintegrationtrigonometric-integrals

$$\int_{5 n \pi}^{\frac{5\pi (n^2 + 1)}{n}}\frac{\frac{8 n^2 x}{25}dx}{((a^2 + b^2) + (a^2 – b^2) cos\frac{2 n x}{5})^2}$$
for $n \in \mathbb{N}\space\space\text{and } a,b >0$

I know this problem has to be solved by properties of definite integrals. so the first thing I tried was.

$$\frac{1}{k}\cdot\int_{ak}^{bk}f\left(\frac{x}{k}\right)dx$$
so by $k=\frac{2n}{5}$

$$\frac{5}{2n}\int_{2\pi n^2 }^{2\pi (n^2 + 1)}\frac{\frac{4nx}{5}dx}{((a^2 + b^2) + (a^2 – b^2) cosx)^2}$$
$$\frac{5}{2n}\cdot\frac{4n}{5}\int_{2\pi n^2 }^{2\pi (n^2 + 1)}\frac{xdx}{((a^2 + b^2) + (a^2 – b^2) cosx)^2}$$
$$2\int_{2\pi n^2 }^{2\pi (n^2 + 1)}\frac{xdx}{(a^2(1+cosx) + b^2(1-cosx))^2}$$

now I have no clue how to solve it further. The function has no period nor did the Weierstrass Substitution work did I mess up at any point? or was my entire method wrong? Kindly help me.

PS: It's a high school-level problem.
$$\text{Solution: }\frac{\pi^2(2n^2+1)(a^2+b^2)}{4a^3b^3}$$

Best Answer

Using the wonderful idea of xpaul, we have $I=(2n^2+1)\pi J$. For completeness, I shall find the easier integral $$ \begin{aligned} J & =\int_0^{2 \pi} \frac{d x}{\left(\left(a^2(1+\cos x)+b^2(1-\cos x\right)\right)^2} \\ & =\frac{1}{2} \int_0^\pi \frac{d x}{\left(a^2 \cos ^2 x+b^2 \sin ^2 x\right)^2} \quad (\textrm{By half-angle formula})\\ & =\int_0^{\frac{\pi}{2}} \frac{d x}{\left(a^2 \cos ^2 x+b^2 \sin ^2 x\right)^2} \\ & =\int_0^{\frac{\pi}{2}} \frac{\sec ^4 x}{\left(a^2+b^2 \tan ^2 x\right)^2} d x \end{aligned} $$ Putting $\tan x =\frac{a}{b}\tan \theta$, we have $$ \begin{aligned} J & =\int_0^{\frac{\pi}{2}} \frac{1+\frac{a^2}{b^2} \tan ^2 \theta}{a^4 \sec ^4 \theta} \frac{a}{b} \sec ^2 \theta d \theta \\ & =\frac{1}{a^3 b} \int_0^{\frac{\pi}{2}} \cos ^2 \theta d \theta+\frac{1}{a b^3} \int_0^{\frac{\pi}{2}} \sin ^2 \theta d \theta \\ & =\frac{1}{a^3 b} \int_0^{\frac{\pi}{2}} \frac{1+\cos 2 \theta}{2} d \theta+\frac{a}{b^3} \int_0^{\frac{\pi}{2}} \frac{1-\cos 2 \theta}{2} d \theta \\ & =\frac{\pi}{4 a^3 b}+\frac{\pi}{4a b^3} \\ & =\frac{\left(a^2+b^2\right) \pi}{4a^3 b^3} \end{aligned} $$ Putting back yields the answer $$ \boxed{I=\frac{\pi^2\left(2 n^2+1\right)\left(a^2+b^2\right)}{4a^3 b^3}} $$

Related Question